Forum des amateurs de maths
Vous souhaitez réagir à ce message ? Créez un compte en quelques clics ou connectez-vous pour continuer.


Aide pour les futurs mathématiciens
 
AccueilAccueil  PortailPortail  RechercherRechercher  Dernières imagesDernières images  S'enregistrerS'enregistrer  Connexion  
Le Deal du moment : -20%
(Adhérents Fnac) Enceinte Bluetooth Marshall ...
Voir le deal
199.99 €

 

  Probleme Août 2015

Aller en bas 
2 participants
AuteurMessage
abdelbaki.attioui
Administrateur
abdelbaki.attioui


Masculin Nombre de messages : 2564
Localisation : maroc
Date d'inscription : 27/11/2005

 Probleme Août 2015 Empty
MessageSujet: Probleme Août 2015    Probleme Août 2015 EmptyDim 02 Aoû 2015, 11:20

Résoudre dans le système :

PGCD(x,y)=1 et x^3(3x+1)=y²(y+1)^3
Revenir en haut Aller en bas
https://mathsmaroc.jeun.fr/
aymanemaysae
Expert grade1



Masculin Nombre de messages : 428
Age : 27
Date d'inscription : 22/01/2014

 Probleme Août 2015 Empty
MessageSujet: Re: Probleme Août 2015    Probleme Août 2015 EmptySam 22 Aoû 2015, 15:53

Bonjour ; en ce qui concerne le problème de Aout, je propose la solution suivante.
Je vais procéder par élimination de cas.
1) x = 0 et y = 0 ; (0 ;0) n’est pas solution car pgcd(0;0) = 0 .
2) x = 0 et y <> 0 ; on a y^2 (y+1)^3 = 0 , donc y = -1 et comme pgcd(0 ;-1) = 1 on a (0 ;-1) solution de l’équation .
3) x <> 0 et y = 0 ; on a x^3 (3 x + 1) = 0 , donc x = - 1/3 , donc aucune solution pour ce cas .
4) x <> 0 et y <> 0 ;
4.1) x = 1 ; on a y^2 (y+1)^3 = 4 ; pas de solution.
4.2) x = - 1 ; on a y^2 (y+1)^3 = 2 ; pas de solution.
4.3) x appartenant à Z/{-1 ; 0 ; 1} et y = 1 ; on a x^3 (3 x + 1) = 8 ; pas de solution.
4.4) x appartenant à Z/{-1 ; 0 ; 1} et y = - 1 ; on a x^3 (3 x + 1) = 0 ; pas de solution.
4.5) x appartenant à Z/{-1 ; 0 ; 1} et y appartenant à Z/{-1 ; 0 ; 1};
Comme pgcd(x ;y) = 1 donc x^3 divise (y + 1)^3 et y^2 divise 3 x + 1, donc il existe (w ;v) appartenant à (Z*)^2 tel que (y + 1)^3 = w x^3 et 3 x + 1 = v y^2 ou bien (y + 1)^3 = u^3 x^3 et 3 x + 1 = v y^2 en posant w = u^3 avec u appartenant à Z*, donc en réinjectant ces deux formes dans l’équation de départ on a : x^3 v y^2 = y^2 u^3 x^3 qui est équivalent à v = u^3.
Donc on a y + 1 = u x et 3 x + 1 = u^3 y^2,
donc |y| + 1 >= |u||x| et 3 |x| + 1 >= |u|^3 |y|^2,
donc |y| + 1 >= |x| et 3 |x| + 1 >= |y|^2, donc |y|^2 <= 3 |y|+ 4
donc |y|^2 - 3 |y|- 4 <= 0 donc (|y| - 4)(|y| + 1) <= 0 donc 2 <= |y| <= 4,
donc y appartient à {-4 ; -3 ; -2 ; 2 ; 3 ; 4}, on trouve que seul y = 4 convient et donne x = 5, donc (5 ; 4) est solution de l’équation.
En conclusion on a {(0 ; -1) , (5 ; 4)} est l’ensemble des solutions.
J’espère que c’est juste, sinon tout un chacun qui proposera une correction ou une solution alternative sera d’une très grande gentillesse.
Revenir en haut Aller en bas
 
Probleme Août 2015
Revenir en haut 
Page 1 sur 1
 Sujets similaires
-
» Probleme Mai 2015
» Probleme Avril 2015
» Probleme juin 2015
» Probleme juillet 2015
» Probleme septembre 2015

Permission de ce forum:Vous ne pouvez pas répondre aux sujets dans ce forum
Forum des amateurs de maths :: Problèmes de la semaine et du mois :: Problème du mois-
Sauter vers: